LSAT and Law School Admissions Forum

Get expert LSAT preparation and law school admissions advice from PowerScore Test Preparation.

 rachue
  • Posts: 140
  • Joined: Jun 22, 2011
|
#1531
Hello. I got this question correct but I want to verify that I used the correct reasoning to get the answer. I chose C because it basically restates the principle in the stimulus, but I was confused as to why the others seem like they have nothing to do with the stimulus. Is it because that, even if they weren't mentioned at all in the stimulus, they still possibly could follow from the stimulus? I've just never encountered an LR question like this! It's weird.
 Nikki Siclunov
PowerScore Staff
  • PowerScore Staff
  • Posts: 1362
  • Joined: Aug 02, 2011
|
#1538
Answer choice (C) is correct, but not for the reasons you state. The question stem is asking you to identify a principle that is NOT logically consistent with the columnist's conclusion (note the word "EXCEPT" at the end of the question stem). This is a Cannot Be True/Principle question.

Essentially, the columnist is arguing that the government should not levy taxes on dangerous activities (hunting, snow skis, parachutes, etc.). Answer choice (C) states the exact opposite: the government should create financial disincentives (such as taxes) to deter participation in such activities. Because this principle contradicts the author's conclusion, it is the correct answer to a Cannot Be True question. The rest of the answer choices state principles that are not in direct opposition to that conclusion, i.e. they are consistent with it. Note that for a statement to be "consistent" with another statement, the two statements cannot contradict one another. As long as a given principle "could be true" in the context of the columnist's conclusion, that principle would be considered "consistent" with it.

Hope this helps.
 rachue
  • Posts: 140
  • Joined: Jun 22, 2011
|
#1543
That does help. Many thanks.
 asteroid
  • Posts: 7
  • Joined: Mar 27, 2012
|
#4156
Need some help with this one:
So the principle is: government should not impose financial impediments on dangerous activities?

I understand why C is the correct answer. But I didn't get how A and E are both consistent with this principle. I first chose A because I thought it's irrelevant.

Thanks.
 Steve Stein
PowerScore Staff
  • PowerScore Staff
  • Posts: 1153
  • Joined: Apr 11, 2011
|
#4162
That's a good question. If two things are consistent, that simply means that they are not inconsistent. They could be completely unrelated, but as long as the answer choices don't directly conflict with the columnists conclusion, they are consistent. Since the question says that the answer choices are all logically consistent EXCEPT, the correct answer choice, as you pointed out, is C, the choice that directly conflicts with the conclusion from the stimulus.

Let me know whether that clears this one up--thanks!

~Steve
 pranavshah7887
  • Posts: 6
  • Joined: Jul 10, 2020
|
#78121
Sorry for seeking a lame clarification here. Could you please let me know whether Can not be true questions type be treated as Must be true except questions? I choose answer A for this, and looking at your response above, I understand that ,since it doesn't directly conflict the claim in the stimulus, it can't be the right answer choice. So am I correct to take it as a rule that 'Can not be true questions' should not be treated as 'MBTX questions' + for a 'Can not be true' we will always have to choose an answer which is in direct contradiction of the claim in the argument?

Regards
Pranav
User avatar
 KelseyWoods
PowerScore Staff
  • PowerScore Staff
  • Posts: 1079
  • Joined: Jun 26, 2013
|
#78870
Hi Pranav!

Cannot Be True questions and Must Be True EXCEPT questions are not quite the same. The logical opposite of "cannot be true" is "could be true." So in a Cannot Be True question, the 4 incorrect answers will be statements that "could be true" or are possible, meaning that they are not directly contradicted by the stimulus. These incorrect answers don't have to be statements that must be true--just statements that cannot be disproven by the stimulus.

The logical opposite of "must be true" is "not necessarily true." In a Must Be True EXCEPT question, the 4 incorrect answers will be statements that "must be true" and the one correct answer will be a statement that is "not necessarily true." A "not necessarily true" statement doesn't have to be something that is directly disproven by the stimulus--just something that cannot be proven by the stimulus.

Hope this helps!

Best,
Kelsey

Get the most out of your LSAT Prep Plus subscription.

Analyze and track your performance with our Testing and Analytics Package.